What Happens to Two Spin-1/2 Particles in a Magnetic Field Over Time?

gasar8
Messages
57
Reaction score
0

Homework Statement


Two particles with spin S_1={1 \over 2} and S_2={1 \over 2} are at t=0 in a state with S=0.
a) Find wave function at t=0 in S_{1z},, S_{2z} basis.
b) Second particle is in a magnetic field B = (\sin\theta,0,\cos\theta), the Hamiltonian is H=\lambda \vec{S_2} \cdot \vec{B}. Find the probability that we find particles at time t in S=1 state.

The Attempt at a Solution


a) |10\rangle ={1 \over \sqrt{2}} \big(|{1 \over 2}-{1 \over 2}\rangle + |-{1 \over 2}{1 \over 2}\rangle\big)

b) If I write Hamiltonian with Pauli matrices, I get:
\lambda \vec{S_2} \cdot \vec{B}=\frac{\lambda \hbar B}{2}<br /> \left(<br /> \begin{array}{cc}<br /> \cos\theta &amp; \sin\theta\\<br /> \sin\theta &amp; -\cos\theta<br /> \end{array}<br /> \right)<br />

Then I wrote Schrödingers equation and got eigenvalues E=\pm \frac{\lambda \hbar B}{2} and eigenvectors (\cot\theta \pm {1 \over \sin\theta},1).
I should be right to this point, right? But now, I have some problems. Is the wave function really: |\psi,0\rangle= |\uparrow\rangle+ \bigg(\cot\theta \pm {1 \over \sin\theta} \bigg) |\downarrow\rangle.
How do I normalize it? And after that, how do I find its time evolution? I tried to put Hamiltonian on both states, so H|\uparrow\rangle and H|\downarrow\rangle, but is it right? Because H is a matrix and |\uparrow\rangle=|{1 \over 2}{1 \over 2}\rangle is vector, so I get another vector?
 
Physics news on Phys.org
gasar8 said:
a) |10⟩=1√2(|12−12⟩+|−1212⟩)|10⟩=12(|12−12⟩+|−1212⟩) |10\rangle ={1 \over \sqrt{2}} \big(|{1 \over 2}-{1 \over 2}\rangle + |-{1 \over 2}{1 \over 2}\rangle\big)
No, that state corresponds to one of ##S=1## states.
gasar8 said:
Second particle is in a magnetic field B=(sinθ,0,cosθ)B=(sin⁡θ,0,cos⁡θ)B = (\sin\theta,0,\cos\theta), the Hamiltonian is H=λ→S2⋅⃗B.H=λS2→⋅B→.H=\lambda \vec{S_2} \cdot \vec{B}. Find the probability that we find particles at time t in S=1S=1S=1 state.
The evolution of the state is calculated by applying the operator ##e^{-iHt/\hbar}## to the initial state ##|00\rangle##, where that time evolution operator only acts on the second particle. So, first find the correct initial state.
 
Ok, so the S=0 must be |00\rangle = {1 \over \sqrt{2}} \big(|{1 \over 2}-{1 \over 2}\rangle - |-{1 \over 2}{1 \over 2}\rangle\big)

blue_leaf77 said:
Time evolution operator only acts on the second particle.
So I must apply Hamiltonian to |S_2 S_{z2}\rangle like H |{1 \over 2}-{ 1 \over 2}\rangle and H |{1 \over 2} {1 \over 2}\rangle?
 
gasar8 said:
So I must apply Hamiltonian to |S2Sz2⟩|S2Sz2⟩|S_2 S_{z2}\rangle like H|12−12⟩H|12−12⟩H |{1 \over 2}-{ 1 \over 2}\rangle and H|1212⟩H|1212⟩H |{1 \over 2} {1 \over 2}\rangle?
Not the Hamiltonian, instead the time evolution operator.
 
So the time evolution operator is something like U=e^{-i{H \over \hbar}t}=e^{\pm i {\lambda B \over 2}t}. But I am not sure how to apply it to an initial state, because there is also first particle in |00\rangle. How do I know it applies only on the second? Is it just because there is a S_2 in Hamiltonian and I need to randomly choose one to be second?
So the time evolution must be:
|\psi(t)\rangle = c_{\uparrow} \ e^{- i {\lambda B \over 2}t} |\uparrow\rangle + c_{\downarrow} \ e^{ i {\lambda B \over 2}t}|\downarrow\rangle.
Do I have to pick these two coefficients from |00\rangle= {1 \over \sqrt{2}} \big(|{1 \over 2}-{1 \over 2}\rangle - |-{1 \over 2}{1 \over 2}\rangle\big), so the |S_2,S_{z2}\rangle = {1 \over \sqrt{2}} \big(|{1 \over 2}-{1 \over 2}\rangle - |{1 \over 2}{1 \over 2}\rangle \big). Is this correct? So the wave function would look like:
|\psi(t)\rangle = - {1 \over \sqrt{2}} e^{- i {\lambda B \over 2}t} |\uparrow\rangle + {1 \over \sqrt{2}} e^{ i {\lambda B \over 2}t}|\downarrow\rangle.

But where is the connection back to both particles and S=1 state? Btw, how do I even know which S=1 state do we need?
 
gasar8 said:
So the time evolution operator is something like U=e−iHℏt=e±iλB2t.U=e−iHℏt=e±iλB2t.
No, you cannot write the last equality. The most right expression is just the eigenvalue of the middle one while the middle one is an operator. There is no way a scalar can be equal to an operator.
gasar8 said:
How do I know it applies only on the second? Is it just because there is a S2S2S_2 in Hamiltonian and I need to randomly choose one to be second?
It's because we know that the Hamiltonian ##H = \lambda \mathbf B \cdot \mathbf S_2## applies to one particle only. Only particle two is placed in a magnetic field, the other particle is not associated with any Hamiltonian. It might be strange that a particle does not have Hamiltonian and hence its time evolution is static but that's the consequence of neglecting the spatial wavefunction and not having this particle interact with magnetic field.
For the purpose of this problem, it's more convenient to separate the kets of the two particle. Thus
$$
|\uparrow \uparrow\rangle \rightarrow |\uparrow\rangle |\uparrow\rangle
$$
If I were to write the complete time evolution operator, it will be ##I\otimes e^{-iHt/\hbar}## where ##I## only acts on the first particle (the first particle is static in time). For example if you apply to ##|11\rangle##, it will be ##(I\otimes e^{-iHt/\hbar})|11\rangle = (I\otimes e^{-iHt/\hbar})(|\uparrow\rangle |\uparrow\rangle) = (I|\uparrow\rangle )(e^{-i\lambda \mathbf B \cdot \mathbf S_2t/\hbar} |\uparrow\rangle)##.
Now, in order to calculate ##e^{-i\lambda \mathbf B \cdot \mathbf S_2t/\hbar} |\uparrow\rangle##, I suggest that you expand the time evolution operator into Taylor series and simplify it using the properties of Pauli matrices. For this matter, you might be interested in the question part of this http://physics.stackexchange.com/questions/41697/what-is-the-spin-rotation-operator-for-spin-1-2.
 
Thread 'Need help understanding this figure on energy levels'
This figure is from "Introduction to Quantum Mechanics" by Griffiths (3rd edition). It is available to download. It is from page 142. I am hoping the usual people on this site will give me a hand understanding what is going on in the figure. After the equation (4.50) it says "It is customary to introduce the principal quantum number, ##n##, which simply orders the allowed energies, starting with 1 for the ground state. (see the figure)" I still don't understand the figure :( Here is...
Thread 'Understanding how to "tack on" the time wiggle factor'
The last problem I posted on QM made it into advanced homework help, that is why I am putting it here. I am sorry for any hassle imposed on the moderators by myself. Part (a) is quite easy. We get $$\sigma_1 = 2\lambda, \mathbf{v}_1 = \begin{pmatrix} 0 \\ 0 \\ 1 \end{pmatrix} \sigma_2 = \lambda, \mathbf{v}_2 = \begin{pmatrix} 1/\sqrt{2} \\ 1/\sqrt{2} \\ 0 \end{pmatrix} \sigma_3 = -\lambda, \mathbf{v}_3 = \begin{pmatrix} 1/\sqrt{2} \\ -1/\sqrt{2} \\ 0 \end{pmatrix} $$ There are two ways...
Back
Top